Đến nội dung

Hoang72

Hoang72

Đăng ký: 21-03-2021
Offline Đăng nhập: Riêng tư
*****

#743851 Chứng minh rằng có một người quen biết tất cả những người còn lại

Gửi bởi Hoang72 trong 26-02-2024 - 10:01

Mình từng đọc bài toán đó ở đây  :D, nhưng giờ mới biết nguồn gốc bài này 




#743522 Chứng minh rằng dãy $(\lfloor x\rfloor)_{n\in\m...

Gửi bởi Hoang72 trong 13-02-2024 - 18:13

Nếu $x_{N_1}=x_{N_1+k}$ thì sao? Khi đó sẽ không tìm được $\alpha$.
Dữ kiện $x_1=q$ hay $q$ hữu tỉ không nguyên chưa được dùng, mặc dù nó không thừa. Ví dụ lấy $q=\frac{5}{2},x_1=\frac{5}{3}$ thì $x_n$ luôn bằng $\frac{5}{3}$, hay lấy $q$ là nghiệm lớn hơn $1$ của phương trình $x^3-x^2-2x+1=0$ thì dãy sẽ tuần hoàn với chu kì $2$.

Hình như, theo em thấy thì hai số hạng bất kỳ của dãy đều phân biệt. Thật vậy, đặt $q = \frac{u}{v},u,v\in\mathbb N^*; \gcd\left(u,v\right) = 1$. Thế thì, bằng quy nạp, ta chứng minh được $x_n = \frac{k_n}{v^n}$, trong đó $k_n$ là số nguyên dương nguyên tố cùng nhau với $v$ xác định bởi $\begin{cases} k_1 = u \\ k_{n+1} = u\cdot \left(k_n\mod v^n\right)\end{cases}$.

Nếu vậy thì lời giải trên là đúng rồi.




#743484 Tồn tại đa thức bậc $ n$ có hệ số nguyên $p ( x )$ sao ch...

Gửi bởi Hoang72 trong 12-02-2024 - 16:52

Mình không rõ số $2a^k + 3$ là biến nào cố định, biến nào không. Nếu $k$ cố định, đặt $q\left(x\right) =p\left(x\right) - 3$, thì ta chỉ cần có $q\in\mathbb Z[x]$ và $q\left(0\right),...,q\left(n\right)$ có dạng $2a^k$. Chú ý theo nội suy Lagrange, $q\left(x\right) = \sum_{i=0}^nP\left(i\right)\prod_{j\neq i}\frac{x-j}{i-j}$, thì ta chọn $P\left(i\right)$ chia hết cho $\prod_{j\neq i}\left(i-j\right)$ là được. Còn nếu $a$ cố định, ta có thể dựa trên khai triển đó: Cho $P\left(x\right) = c\cdot\sum_{i=0}^nt^i\binom{x}{i}$. Khi đó với $0\leq z\leq n$, ta có $P\left(z\right)= c\left(t+1\right)^z$. Ta chọn $t = a^u - 1,c = 2a^v$ thì ta chỉ cần đảm bảo $P\in\mathbb Z[x]$. Chọn $u,v$ phù hợp và đủ lớn sao cho $n!\mid c\cdot t$, ta có điều phải chứng minh.




#743482 Tồn tại đa thức bậc $ n$ có hệ số nguyên $p ( x )$ sao ch...

Gửi bởi Hoang72 trong 12-02-2024 - 16:21

E có một chút thắc mắc về bổ đề ạ ví dụ như $P(x)=\frac{1}{n}x(x-1)(x-2)(x-3)\ldots (x-(n-1))$ là đa thức bậc $n$ và thoả mãn $P(0),P(1),P(2),\ldots,P(n)$ là các số nguyên mà e thấy $P(x) \not \in \mathbb Z[x]$ ạ, mong được giải đáp ạ. E cảm ơn nhìu ạ.

À xin lỗi bạn, hồi đó mình nhầm á  :luoi: , bổ đề chỉ cho ta $P\left(x\right)\in\mathbb Z,\forall x\in\mathbb Z$ thôi




#743480 CM tồn tại 2 ô vuông chung cạnh có hiệu hai số lơn hơn hoặc bằng $n +1...

Gửi bởi Hoang72 trong 12-02-2024 - 16:17

Bài này theo mình nghĩ, là yêu cầu chứng minh tồn tại hai ô chung cạnh có hiệu hai số lớn hơn hoặc bằng $n$.

Xét $k$ là số nguyên dương nhỏ nhất sao cho các số $1,2,...,k$ hoặc xuất hiện trên mọi hàng, hoặc xuất hiện trên mọi cột.

Không mất tính tổng quát, coi $k$ số này xuất hiện trên mọi hàng.

NX. Mọi hàng đều chứa ít nhất một ô không chứa một trong các số $1,2,...,k$.

Chứng minh. Giả sử tồn tại hàng $i$ trái với tính chất trên. Khi đó các số $1,2,...,k$ cũng xuất hiện trên mọi cột, mà các số $1,2,..,k-1$ không xuất hiện trên mọi cột nên $k$ thuộc hàng $i$. Chứng tỏ các số $1,2,...,k-1$ vẫn xuất hiện trên mọi hàng, mâu thuẫn.

Như vậy, với hàng $i$ bất kì, ta tìm được hai ô chứa hai số $a_i,b_i$ sao cho $a_i\in\left\{1,2,...,k\right\}$ và $b_i > k$. Chú ý $b_1,b_2,...,b_n$ là các số lớn hơn $k$ nên tồn tại một số $b_j$ không nhỏ hơn $k + n$.

Thế thì $b_j - a_j \geq n$, và $b_j,a_j$ là hai số ở hai ô chung cạnh. (đpcm)

 




#742952 Có bao nhiêu cách sắp xếp $2n$ quả bóng trên một đường thẳng

Gửi bởi Hoang72 trong 10-01-2024 - 10:24

Nhận thấy nếu đổi chỗ quả bóng đen $a$ và quả bóng đen $b$, đồng thời đổi chỗ quả bóng trắng $a$ và quả bóng trắng $b$ thì điều kiện bài toán vẫn được đảm bảo.

Do đó ta không mất tính tổng quát, coi các quả bóng đen được sắp xếp theo thứ tự tăng dần (quả bóng đen $i$ đứng trước quả bóng đen $i+1$)

Khi đó, các quả bóng trắng được sắp xếp theo thứ tự giảm dần.

Gọi $u_n$ là số cách sắp xếp thoả mãn các điều kiện trên. 

+) Nếu quả bóng đen $n$ đứng trước quả bóng trắng $n$ thì ta có duy nhất cách xếp thoả mãn là đen $1$, đen $2$,..., đen $n$, trắng $n$,..., trắng $2$, trắng $1$.

+) Nếu quả bóng đen $n - 1$ đứng trước quả bóng trắng $n$ thì sử dụng điều kiện b), ta có quả bóng đen $n$ đứng trước các quả bóng trắng $n-1,...,1$. Như vậy ta cũng có duy nhất cách xếp thoả mãn là đen $1$, đen $2$,..., đen $n-1$, trắng $n$, đen $n$, trắng $n-1$,..., trắng $1$.

+) Nếu quả bóng trắng $n$ đứng trước quả bóng đen $i$ mà đứng sau quả bóng đen $i-1$ (coi quả bóng đen $0$ đứng trước mọi quả bóng) thì quả bóng đen $n$ đứng trước các quả bóng trắng $1,...,i-1$, và đứng sau các quả bóng trắng $i,...,n$. Rõ ràng các quả bóng $1,...,i-1$ và $n$ đã chọn được vị trí. Chú ý các quả bóng trắng $i,...,n - 1$ cũng đứng sau quả bóng đen $i-1$. Như vậy ta cần xếp các quả bóng trắng $i,...,n-1$ vào các vị trí cố định của các quả bóng đen $i,...,n-1$, số cách xếp là $u_{n - i}$.

Dẫn đến $u_n = 2 + u_1 + u_2 + ... + u_{n-1}$. Bằng quy nạp, ta chứng minh được $u_n = 2^{n},\forall n\in\mathbb N^*$.

Kết hợp với $n!$ hoán vị của các quả bóng đen, ta có tất cả $2^{n} . n!$ cách xếp.




#742697 Có bao nhiêu cách đi khác nhau từ (0,0) đến (6,6).

Gửi bởi Hoang72 trong 25-12-2023 - 08:23

Em xin thử một hướng tiếp cận để đi đến công thức tổng quát cho số cách đi từ $\left(0,0\right)$ đến $\left(n,n\right)$.

Gọi số bước từ $\left(x,y\right)$ đến $\left(x + 1,y\right)$ là $a$; số bước từ $\left(x,y\right)$ đến $\left(x,y+1\right)$ là $b$; số bước từ $\left(x,y\right)$ đến $\left(x+2,y+1\right)$ là $c$.

Khi đó, ta có $a + 2c = b + c = n\Rightarrow b = a+c$

$\Rightarrow a+b+c=2b$.

Ngoài ra, với mỗi $a,b,c$ ta có số cách di chuyển là $\frac{\left(a+b+c\right)!}{a!b!c!}$.

Do đó, số cách đi từ $\left(0,0\right)$ đến $\left(n,n\right)$ là $$u_n = \sum_{b = \left\lceil n/2\right\rceil}^{n}\frac{\left(2b\right)!}{\left(2b-n\right)!b!\left(n-b\right)!} = \sum_{b = \left\lceil n/2\right\rceil }^n\binom{n}{b}\binom{2b}{n}$$

Tuy nhiên, khai triển ta thấy $\left[\left(x+1\right)^2+1\right]^n = \sum_{k = 0}^n \binom{n}{k}\left(x+1\right)^{2k}$.

Như vậy, $u_n$ thực chất là hệ số của $x^n$ trong đa thức $\left[\left(x + 1\right)^2 + 1\right]^n = \left(x + i + 1\right)^n\left(x + 1 - i\right)^n$

$\Rightarrow u_n = \sum_{k=0}^n \left(i+1\right)^k \left(1 - i\right)^{n - k} \binom{n}{k}^2 = \left(1-i\right)^n \sum_{k=0}^n i^k\binom{n}{k}^2$.

Mặt khác, ta chứng minh được (tham khảo ở đây) $$P_n\left(\frac{z+1}{z-1}\right) = \frac{1}{\left(z-1\right)^n}\sum_{k=0}^n z^k\binom{n}{k}^2$$

với $P_n\left(x\right)$ là đa thức Legendre thứ $n$.

Dẫn đến $u_n =\left(-1\right)^n\left(1-i\right)^{2n}P_n\left(\frac{i+1}{i-1}\right) =\left(2i\right)^nP_n\left(-i\right)$.

Ta lại có công thức truy hồi $$\begin{cases} P_0\left(x\right) = 1, P_1\left(x\right) = x \\ \left(n+1\right)P_{n+1}\left(x\right) = \left(2n+1\right)xP_n\left(x\right) - nP_{n-1}\left(x\right)\end{cases}$$

Như vậy, dễ dàng thu được $$\begin{cases} u_0 = 1,u_1 = 2 \\ \left(n+1\right)u_{n+1} = 2\left(2n+1\right)u_n + 4nu_{n-1}\end{cases}$$

Ta tính được các số hạng đầu là $1,2,8,32,136,592,2624,11776$.

Tuy nhiên, công thức trên vẫn mất $O\left(n\right)$ thời gian khi tính $u_n$ cụ thể nào đó.

Hy vọng mọi người sẽ có sự cải tiến nào đó.




#741313 Chứng minh rằng các số hạng ở dãy $\left ( a_{n} \ri...

Gửi bởi Hoang72 trong 05-09-2023 - 23:36

Ta có $b_{n+1} = a_nb_n = a_na_{n-1}b_{n-1} = ... = a_n...a_1b_1 = a_n...a_1,\forall n\in\mathbb N^*$

$\Rightarrow b_n = \prod_{i=1}^{n-1} a_i,\forall n\in\mathbb N^*$ (với quy ước tích rỗng có giá trị bằng $0$)

Như vậy, $a_{n+1} = a_n + \prod_{i=1}^{n-1} a_i,\forall n\in\mathbb N^*$. 

Đến đây chỉ cần sử dụng quy nạp chứng minh $\gcd\left(a_n,a_1...a_{n-1}\right)=1$ ta sẽ suy ra được các số hạng của dãy $\left\{a_n\right\}$ đôi một nguyên tố cùng nhau.




#740315 Chứng minh $NM$ tiếp xúc với $(I)$

Gửi bởi Hoang72 trong 02-07-2023 - 07:12

Kẻ đường kính $DD'$ của $(I)$. Cho $AD'$ cắt lại $(I)$ tại $H$.

Theo kết quả quen thuộc, ta có $IM \parallel AD'\Rightarrow IM \parallel D'H$

$\Rightarrow IM \perp DH$.

Mà $MD$ là tiếp tuyến của $(I)$ nên $MH$ cũng là tiếp tuyến của $(I)$

Lấy $J$ là trung điểm của $EF$.

Ta có $AD'.AH = AE^2= AJ.AI\Rightarrow$ Tứ giác $IJD'H$ nội tiếp

$\Rightarrow \angle AHJ = \angle AID' = \angle ATJ$ (có các cạnh tương ứng vuông góc)

$\Rightarrow$ Tứ giác $ATHJ$ nội tiếp

$\Rightarrow \angle AHT = \angle AJT = 90^\circ\Rightarrow D,H,T$ thẳng hàng.

Gọi $\left\{N'\right\}  = MH \cap AT$. Ta có $\angle N'HD' = \angle HDD' = 90^\circ - \angle ATH = \angle HAT$

$\Rightarrow \Delta N'HA$ cân tại $N'$.

Kết hợp với điều kiện tam giác $AHT$ vuông tại $H$, ta có $N'$ là trung điểm AT$

$\Rightarrow N'\equiv N\Rightarrow MN$ tiếp xúc với $(I)$ tại $H$.

 

File gửi kèm




#740181 Chứng minh $n$ là số chính phương biết $\frac{1}{\sq...

Gửi bởi Hoang72 trong 24-06-2023 - 20:18

Bổ đề 1: Cho $r$ là số hữu tỉ thoả mãn $Q(x) = x^7 - r$ khả quy. Khi đó $\sqrt[7]{r}\in\mathbb Q$.

Chứng minh: Giả sử $Q(x) = g(x)h(x)$, với $g,h\in\mathbb Q[x]; \deg g,\deg h > 0$.

Gọi $z_1,z_2,...,z_k$ là các nghiệm (phức) của $g$, $0 < k < 7$.

Khi đó $|z_i|^7 = |r|,\forall i = \overline{1,k}$

$\Rightarrow |z_i| = \sqrt[7]{|r|}\Rightarrow \sqrt[7]{|r^k|} = |z_1z_2...z_k|\in\mathbb Q$.

Tuy nhiên $7$ là số nguyên tố nên từ đây dễ dàng suy ra $\sqrt[7]{|r|}\in\mathbb Q$

$\Rightarrow \sqrt[7]{r}\in\mathbb Q$.

Bổ đề 2: Cho $r\in\mathbb Q, \sqrt[7]{r}\not\in\mathbb Q$ và bộ số hữu tỉ $(a_0,a_1,...,a_6)$ thoả mãn $a_0 + a_1\sqrt[7]{r} + a_2\sqrt[7]{r^2} + ... + a_6\sqrt[7]{r^6} = 0$. Khi đó $a_0=a_1=...=a_6 = 0$.

Chứng minh: Từ Bổ đề 1 ta dễ dàng rút ra $x^7 - r$ là đa thức tối tiểu của $\sqrt[7]{r}$.

Mặt khác đa thức $a_0 + a_1x + ... + a_6x^6$ nhận $\sqrt[7]{r}$ là nghiệm và có bậc nhỏ hơn $7$. Như vậy đa thức này là đa thức không hay $a_ i = 0,\forall i = \overline{0,6}$.

Trở lại bài toán: Xét $(x,y)$ là một nghiệm nguyên dương của phương trình đã cho.

Nhận thấy $\left(\frac{1}{n} - \frac{1}{\sqrt[7]{y}}\right)^3\in\mathbb Q$ nên nếu $\sqrt[7]{y}\not\in\mathbb Q$ thì từ Bổ đề 2, rút ra $\frac{1}{n} = 0$, vô lý.

Do đó $\sqrt[7]{y}\in\mathbb Q\Rightarrow y = b^7,b\in\mathbb N^*$.

Dẫn đến $\sqrt[3]{x}\in\mathbb Q$, nên $x = a^3,a\in\mathbb N^*$.

Suy ra số nghiệm nguyên dương của phương trình đã cho cũng chính bằng số nghiệm nguyên dương của phương trình (ẩn $a,b$) $$\frac{1}{a} +\frac{1}{b} = \frac{1}{n}$$

Đặt $n = p_1^{k_1}...p_l^{k_l}$. Dễ dàng chỉ ra, số nguyên nguyên dương của phương trình trên là số ước của $n^2$, tức là $$m = (2k_1 + 1)...(2k_l + 1)$$

Vì $m - 1$ là số chính phương, đặt $m-1 = k^2,k\in\mathbb N^*$.

Khi đó $k^2 + 1 = (2k_1 + 1)...(2k_l + 1)$. Mà $k^2+1$ không có ước dạng $4j + 3$ nên $2k_i+1\equiv 1\pmod 4$

$\Rightarrow k_i \vdots 2,\forall i = \overline{1,l}$.

Vậy $n$ là số chính phương.




#740180 $\left | \frac{a-b}{c} +\frac{b-c}{a}+\frac{c-a}{b}...

Gửi bởi Hoang72 trong 24-06-2023 - 19:42

Không biết cách này có ngộ nhận đoạn nào không:

Kí hiệu $D = \left(\frac{1}{2},1\right)$.

Với mọi bộ $(a,b,c)\in D^3$ thoả mãn $a\leq b\leq c$, đặt $f(a,b,c) = \frac{(a-b)(b-c)(c-a)}{abc}$. 

Thế thì ta cần chứng minh $f(a,b,c)\leq  \left(1 - \frac{\sqrt{2}}{2}\right)^2,\forall (a,b,c)\in D^3; a\leq b\leq c$.

Nhận thấy nếu thay bộ $(a,b,c)$ bởi bộ $(x,y,z)$ thoả mãn $c-z = b-y = a - x$ và $x = \frac{1}{2}$ thì $f(a,b,c) \leq f(x,y,z)$.

Do đó không mất tính tổng quát giả sử $a = \frac{1}{2}$. 

Thế thì ta cần chứng minh $$\frac{(2b-1)(c-b)(2c-1)}{bc} \leq 3 - 2\sqrt{2}$$

Hơn nữa, nhận thấy hàm số $g(t) = \frac{2t-1}{t}$ đồng biến trên $\mathbb R^+$ nên nếu tăng giá trị của $b,c$ lên cùng một đơn vị thì $f\left(\frac{1}{2},b,c\right)$ cũng tăng.

Như vậy không mất tính tổng quát coi $c=1$. Ta quy về chứng minh $$\frac{(2b-1)(1-b)}{b} \leq 3 - 2\sqrt{2},\forall b\in D$$

Tuy nhiên bất đẳng thức trên tương đương $\left(\sqrt{2}b - 1\right)^2 \geq 0$, đúng với mọi $b\in\mathbb R$.

Vậy ta có đpcm




#739121 Chứng minh $\sum \frac{2a^2+ab}{(a+b+\sqrt{ac})^2} \...

Gửi bởi Hoang72 trong 09-05-2023 - 11:36

Áp dụng bất đẳng thúc Cauchy - Schwarz ta có: $(a+b+\sqrt{ac})^2 \leq (a+b+a)(a+b+c) = (2a+b)(a+b+c)$

$\Rightarrow \sum\frac{2a^2+ab}{(a+b+\sqrt{ac})^2}\geq \sum\frac{a}{a+b+c} = 1$. (đpcm)




#739120 CM $\sum \sqrt{\frac{2ab(a^2-ab+b^2)}...

Gửi bởi Hoang72 trong 09-05-2023 - 11:34

Nhận thấy: $3(a^4 + b^4) - 2(a^2+ab+b^2)(a^2 - ab+b^2) = (a^2-b^2)^2\geq 0$

$\Rightarrow \sum \sqrt{\frac{2ab(a^2-ab+b^2)}{a^4+b^4}} \leq \sum\sqrt{\frac{3ab}{a^2+ab+b^2}}$.

Như vậy ta chỉ cần chứng minh $$\sum\sqrt{\frac{3ab}{a^2+ab+b^2}}\leq \frac{\left(\sqrt{a}+\sqrt{b}+\sqrt{c}\right)^2}{a+b+c}\text{ }(*)$$ 

Ta có $(*)\Leftrightarrow \sum \left(1 - \sqrt{\frac{3ab}{a^2+ab+b^2}}\right)\geq \frac{\sum\left(\sqrt{a} - \sqrt{b}\right)^2}{a+b+c}$ 

$\Leftrightarrow \sum\frac{(a-b)^2}{a^2+ab+b^2 + \sqrt{3ab\left(a^2+ab+b^2\right)}}\geq \sum\frac{(a-b)^2}{(a+b+c)\left(\sqrt{a} + \sqrt{b}\right)^2}$.

Do đó xét hiệu: \begin{align*}(a+b)\left(\sqrt{a}+\sqrt{b}\right)^2 -\left(a^2+ab+b^2\right) - \sqrt{3ab\left(a^2+ab+b^2\right)} &> 2\sqrt{ab}(a+b) - \sqrt{3ab\left(a^2+2ab+b^2\right)}  \\&= \left(2 - \sqrt{3}\right)\sqrt{ab}(a+b)>0 \end{align*}

$\Rightarrow (a+b+c)\left(\sqrt{a}+\sqrt{b}\right)^2 > \left(a^2+ab+b^2\right) + \sqrt{3ab\left(a^2+ab+b^2\right)}$.

Thiết lập 2 bất đẳng thức tương tự ta có đpcm.

 




#738648 Đề thi chọn đội tuyển Olympic quốc tế (TST) năm 2023

Gửi bởi Hoang72 trong 17-04-2023 - 08:32

Bài 4:

Dễ thấy công thức tổng quát của $(x_n)$ là: $x_n = \left(\frac{\sqrt{a^2+4b} +a}{2}\right)^n + \left(\frac{-\sqrt{a^2+4b} +a}{2}\right)^n$

Trước tiên, ta có một số nhận xét sau: 

Nhận xét 1: $(x_n, b) = 1,\forall n\in\mathbb N^*$.

Chứng minh: Từ giả thiết ta có ngay $(x_0,b)=(x_1,b) = 1$. 

Mặt khác $x_{n+2} \equiv ax_{n+1}\pmod b,\forall n\in\mathbb N$ nên bằng quy nạp, dễ dàng suy ra đpcm.

Nhận xét 2: $\forall m,n\in\mathbb N^*: x_n\mid x_m\Rightarrow \begin{cases} n\mid m \\ v_2(n) = v_2(m)\end{cases}$.

Chứng minh: Đặt $t$ là số dư của $m$ khi chia cho $2n$.

Nhận thấy: $x_m = x_nx_{m-n} - (-b)^n\left(\left(\frac{\sqrt{a^2+4b} +a}{2}\right)^{m-2n} + \left(\frac{-\sqrt{a^2+4b} +a}{2}\right)^{m-2n}\right)$.

Do đó nếu $m\geq 2n$ thì $x_n\mid (-b)^nx_{m-2n} \Rightarrow x_n\mid x_{m-2n}$.

Cứ tiếp tục như thế, ta có: $x_n\mid x_t\Rightarrow n\leq t$.

Mặt khác, ta có: $x_t = x_nx_{t-n} - (-b)^{t-n}x_{2n - t}\Rightarrow x_n\mid x_{2n-t} \Rightarrow n\leq 2n - t\Rightarrow n \geq t$.

Như vậy, $n = t$. Suy ra $2n\mid m - n$, hay $\begin{cases} n\mid m \\ v_2(n) = v_2(m)\end{cases}$.

Trở lại bài toán:

a) Vì $2\mid a$, $2\nmid b$ nên $x_{n+2}\equiv bx_n\equiv x_n\pmod 2,\forall n\in\mathbb N$

$\Rightarrow 2\mid x_n,\forall n\in\mathbb N$.

Từ đó, $4\mid ax_n$ nên $x_{n+2}\equiv bx_n\pmod 4,\forall n\in\mathbb N$. $(*)$

Từ $(*)$ dễ có $v_2(x_n) = 1,\forall n\in\mathbb N, 2\mid n$.

Đồng thời, bằng quy nạp, dễ dàng chứng minh được $v_2(x_n) = v_2(a),\forall n\in\mathbb N, 2\nmid n$.

Giả sử tồn tại $m,n,p\in\mathbb N^*$ mà $\frac{x_m}{x_nx_p}\in\mathbb Z$.

Nếu $n$ hoặc $p$ lẻ thì $v_2(x_nx_p)\geq v_2(a) + 1\geq v_2(x_m)$, vô lí,

Do đó $n,p$ đều chẵn. Hay $4\mid x_m\Rightarrow 2\nmid m$.

Mặt khác, $x_n\Rightarrow x_m\Rightarrow n\mid m$ (Từ Nhận xét 2).

Điều này rõ ràng vô lí. Ta có đpcm.

b) Giả sử tồn tại $m,n,p\in\mathbb N^*$ thoả mãn $\begin{cases} 2\mid mnp \\ \dfrac{x_m}{x_nx_p}\text{ là số chính phương}\end{cases}$.

Thế thì, $\begin{cases} x_n\mid x_m \\ x_p\mid x_m\end{cases}\Rightarrow v_2(m)=v_2(n)=v_2(p)$

$\Rightarrow m,n,p$ chẵn.

Nhận thấy với mọi $k\in\mathbb Z$ thì: $x_{2k} = x_k^2 - 2(-b)^k$.

Vì $b$ lẻ nên suy ra $x_{2k}\equiv 2,3\pmod 4$.

Suy ra $x_m,x_n,x_p\equiv 2,3\pmod 4$.

TH1: $x_m\equiv 3\pmod 4$: Khi đó $2\nmid x_n,x_p\Rightarrow x_n,x_p\equiv 3\pmod 4$

$\Rightarrow \frac{x_m}{x_nx_p}\equiv 3\pmod 4$, vô lí.

TH2: $x_m\equiv 2\pmod 4$: Đặt $y_k = x_{2k},\forall k\in\mathbb N$.

Khi đó: $\begin{cases} y_0 = 2 \\ y_1 = a^2 + 2b \\ y_{n+2} = (a^2 + 2b)y_{n+1} - b^2y_n,\forall n\in\mathbb N\end{cases}$.

Vì $a,b$ lẻ nên $a^2+2b\equiv 3\pmod 4$. Suy ra dãy tuần hoàn mod $4$ của $(y_n)$ là: $2,3,3,2,3,3,...$.

$\bullet$ $a^2+2b\equiv 3\pmod 8$: Khi đó dãy tuần hoàn mod $8$ của $(y_n)$ là: $2,3,7,2,7,3,2,3,7,...$.

$\bullet$ $a^2+2b\equiv 7\pmod 8$: Khi đó dãy tuần hoàn mod $8$ của $(y_n)$ là: $2,7,7,2,7,7,...$.

Do đó $y_n\equiv 2\pmod 4\Leftrightarrow y_n\equiv 2\pmod 8$.

Điều này dẫn đến $x_m\equiv 2\pmod 8$.

Và $x_n,x_p$ hoặc chia $4$ dư $3$, hoặc chia $8$ dư $2$.

Xét các trường hợp ta thấy $\frac{x_m}{x_nx_p}\equiv 2,3\pmod 4$, vô lí.

Ta có điều phải chứng minh.




#738333 Khai triển Viète và Khai triển Tchebychev

Gửi bởi Hoang72 trong 04-04-2023 - 11:02

Bài toán 1: Với mọi $x\geq -1$, xét khai triển: $$\frac{\left(\sqrt{x+1}+1\right)^n+\left(1 - \sqrt{x+1}\right)^n}{2} = \sum_{i=0}^{\left \lfloor \frac{n}{2}\right\rfloor} \binom{n}{2i}(x+1)^{i} = \sum_{i=0}^{\left \lfloor \frac{n}{2}\right\rfloor}\sum_{j = 0}^i \binom{n}{2i}\binom{i}{j}x^j$$

Do đó VT thực chất là hệ số của $x^k$ trong khai triển $$u_n(x) = \frac{\left(\sqrt{x+1}+1\right)^n+\left(1 - \sqrt{x+1}\right)^n}{2}$$

Dễ dàng nhận thấy $u_{n+2}(x) = 2u_{n+1}(x) + xu_n(x)$, do đó đặt $F(n,t)$ là hệ số của $x^t$ trong $u_n(x)$ thì \begin{equation}\label{(1)} f(n,t) = 2f(n-1,t) + f(n-2,t-1)\end{equation}

Đặt $T(n,t) = \frac{F(n,t)}{(-1)^t}$ thì $(1)$ trở thành $$T(n,t) =  2T(n-1,t) - T(n-2,t-1)$$

Do đó hàm $T$ ở đây trùng với hàm $T$ của bài viết. Vậy $\boxed{\displaystyle \sum_{j=0}^{\left\lfloor\frac{n}{2}\right\rfloor} {n\choose 2j} {j\choose k}=\frac{2^{n-1-2k}n}{n-k} {n-k\choose k}}$.

P/s: Ban đầu em có đọc post kia của thầy Thành trước nhưng chỉ làm được đến $(1)$. Đọc post này mới nhận ra hai hàm này gần như giống nhau.